Difference between revisions of "1987 OIM Problems/Problem 4"
(solution) |
|||
Line 1: | Line 1: | ||
== Problem == | == Problem == | ||
− | We define the | + | We define the sequence <math>p_n</math> as follows: <math>p_1=2</math>, and for all <math>n\ge2</math>, <math>p_n</math> is the greatest prime divisor of the expression: |
− | <cmath> | + | <cmath>p_1p_2p_3\cdots p_{n-1} +1</cmath> |
− | Prove that <math>p_n</math> | + | Prove that <math>p_n\ne5</math>. |
~translated into English by Tomas Diaz. ~orders@tomasdiaz.com | ~translated into English by Tomas Diaz. ~orders@tomasdiaz.com | ||
+ | |||
+ | ~additional modifications by [https://artofproblemsolving.com/wiki/index.php/User:Eevee9406 eevee9406] | ||
== Solution == | == Solution == | ||
− | {{ | + | We get <math>p_2=3</math> from the definition; thus for all <math>k\ge 3</math>, we must have <math>2,3\nmid p_k</math>. But if we assume for the sake of contradiction that there exists <math>n</math> such that <math>p_n=5</math>, then <math>p_1p_2\cdots p_{n-1}=5^m-1</math> for some positive integer <math>m</math> because <math>5</math> is the minimal prime divisor of <math>p_1p_2\cdots p_{n-1}</math> since <math>2</math> and <math>3</math> are not prime divisors, but <math>5</math> must be the maximal prime divisor by definition. However, taking mod <math>4</math> reveals that the LHS is divisible by <math>2</math> but not <math>4</math> whereas the RHS is divisible by <math>4</math>, a contradiction. |
+ | |||
+ | ~ [https://artofproblemsolving.com/wiki/index.php/User:Eevee9406 eevee9406] | ||
== See also == | == See also == | ||
https://www.oma.org.ar/enunciados/ibe2.htm | https://www.oma.org.ar/enunciados/ibe2.htm |
Latest revision as of 21:43, 26 August 2025
Problem
We define the sequence as follows:
, and for all
,
is the greatest prime divisor of the expression:
Prove that
.
~translated into English by Tomas Diaz. ~orders@tomasdiaz.com
~additional modifications by eevee9406
Solution
We get from the definition; thus for all
, we must have
. But if we assume for the sake of contradiction that there exists
such that
, then
for some positive integer
because
is the minimal prime divisor of
since
and
are not prime divisors, but
must be the maximal prime divisor by definition. However, taking mod
reveals that the LHS is divisible by
but not
whereas the RHS is divisible by
, a contradiction.